Matrice semblable M(n;Q)

Bonjour

$A,B \in GL(n;\Q)$ et semblables dans $M(n;\Q)$ . On suppose qu’il existe $t\geq 1$ entier tel que $A^{t+1}B=BA^{t}.$
Montrer que $A=B=I_{n}.$

Merci.

Réponses

  • Bonsoir,
    C'est une question ou un exercice ?
    On peut commencer par étudier le cas "$A$ inversible".
  • D'après l'énoncé, ceci fait, on peut s'arrêter...
  • On montre que $A-Id$ est nilpotente et donc, comme $B$ est semblable à $A$, que $B-Id$ est nilpotente.
    On se place dans une base où $A=Id+J$ avec $J$ nulle partout sauf éventuellement juste au dessus de la diagonale.
    Soit $r \in \N$ tel que $J^r\neq 0$ et $J^{r+1}=0$.
    $A^k=Id+kJ+ \dots +C^r_kJ^r$
    Donc $\|A^k\|\geq C^r_k$ dès que $k\geq r$. Et $C^r_k$ est un polynôme en $k$ de degré $r$. Donc $\geq \delta k^r$ pour $k$ grand avec $\delta >0$.
    Il existe $\alpha$ tel que $\|A^k\| \leq \alpha k^r$ pour tout $k>0$, car les coefficients de $A^k$ sont des polynômes en $k$ de degré au plus $r$.
    $B^{-1}-Id$ est nilpotente car c'est $B^{-1}(Id-B)$ et $Id-B$ est nilpotente.
    Donc $B^{-1}=Id+N$ avec $N$ nilpotente. Donc $N^n=0$
    De même $\|B^{-k}\|\leq \beta k^n$.
    $\|A^{(t+1)^k}B^k\|\|B^{-k}\|\geq \|A^{(t+1)^k}\|$
    Donc
    $\|A^{(t+1)^k}B^k\|\geq \delta (t+1)^{kr}/(\beta k^n)$

    $\|B^kA^{t^k}\| \leq \alpha t^{kr} \gamma k^r$ où $\gamma$ est l'analogue de $\alpha$ pour $B$ qui est nilpotente de même degré, car $A$ et $B$ sont semblables.

    Or $A^{(t+1)^k}B^k=B^kA^{t^k}$
    Donc $\delta (t+1)^{kr}/(\beta k^n) \leq \alpha t^{kr} \gamma k^r$
    Lorsque $k$ tend vers l'infini, on obtient une contradiction si $r>0$.
    Donc $r=0$ et $J=0$, donc $A=Id$ et $B$ semblable à $A$ est aussi l'identité.
  • Pour montrer $A-Id$ nilpotente, on montre que toutes les valeurs propres de $A$ sont $1$, en considérant le fait que, pour tout $k$, $A^{(t+1)k}$ est semblable à $A^{tk}$ par $B$ la matrice de passage. Puis on prend la trace.
  • Pourquoi $A_{22}$ et $B_{22}$ sont-ils semblables ? Il faut qu'ils soient semblables pour appliquer l'hypothèse de récurrence.
  • Marco: je ne comprends pas comment tu montres que $A-I_n$ est nilpotente: par exemple pour $A=\begin{pmatrix} 1 & 0 & 0 \\ 0 & j & 0 \\ 0 & 0 & j^2\end{pmatrix}$ ($j=e^{2i\pi/3}$) et $B=\begin{pmatrix} 1 & 0 & 0 \\ 0 & 0 & 1 \\ 0 & 1 & 0 \end{pmatrix}$ on a bien $BA^2B^{-1}=A$.

    Avec les hypothèses de l'énoncé on peut en revanche montrer que les valeurs propres de $A$ sont toutes de module $1$ et même des racines de l'unité: puisque $A^{t+1}$ est semblable à $A^t$, on peut trouver une application $T:Sp(A)\to Sp(A)$ (où $Sp(A)$ désigne le spectre de $A$) telle que $T(\zeta)^{t+1}=\zeta^t$ pour tout $\zeta\in Sp(A)$. Pour un $\zeta\in Sp(A)$ donné, il existe $l>m\geqslant 0$ tels que $T^l(\zeta)=T^m(\zeta)$ puis, en élevant à la puissance $(t+1)^l$, $\zeta^{t^l}=\zeta^{t^m (t+1)^{l-m}}$ et comme $t^l\neq t^m (t+1)^{l-m}$ il s'ensuit que $\zeta$ est une racine de l'unité.

    Après, je crois que le reste de ton raisonnement s'adapte facilement: on écrit la décomposition de Dunford $A=D+N$ et, si $r$ est l'indice de nilpotence de $N$, pour tout $k\geqslant 0$ on a $A^k=\sum_{i=0}^{r-1} \binom{k}{i} D^{k-i} N^i$. Comme $D$ est diagonalisable à valeurs propres de module $1$, la suite de ses puissances $(D^k)_{k\in \mathbb{Z}}$ est bornée. Il s'ensuit que la suite des normes $(\lVert A^k\rVert)_{k\geqslant 1}$ croît essentiellement comme $k^{r-1}$ c'est-à-dire qu'il existe des constantes $c_1,c_2>0$ telles que $c_1k^{r-1}\leqslant \lVert A^k\rVert\leqslant c_2k^{r-1}$ pour tout $k\geqslant 1$. Puisque $B$ est semblable à $A$, une propriété similaire est valable pour $B$ et aussi pour $B^{-1}$. Les normes de $B^k A^{t^k}B^{-k}$ et $A^{(t+1)^k}$ sont égales pour tout $k\geqslant 1$ mais la première est majorée à une constante près par $t^{k(r-1)}k^{2(r-1)}$ tandis que la deuxième croît comme $(t+1)^{k(r-1)}$ ce qui n'est possible que si $r=1$ c'est-à-dire $N=0$.

    Ainsi, $A$ est diagonalisable donc d'ordre fini (puisque ses valeurs propres dont des racines de l'unité) et il en va de même de $B$. Soit $p\geqslant 1$ l'ordre de $A$ (i.e. le plus petit entier positif tel que $A^p=I_n$). On a alors $B^p=I_n$ et $B^p A^{t^p}B^{-p}=A^{(t+1)^p}$ donc $A^{(t+1)^p-t^p}=I_n$ et par conséquent $p$ divise $(t+1)^p-t^p$. Cette dernière propriété implique $p=1$: autrement on considère le plus petit diviseur premier $q$ de $p$ et alors $q$ ne divise pas $(t+1)^p-t^p$ car $p$ est premier avec $q-1$.
  • Pea: oui, tu as raison, je me suis trompé pour $A-Id$ nilpotente.
  • @merci à zephir, math coss , marco, side pour vos posts .

    C’est un exo style oral ENS ULM
Connectez-vous ou Inscrivez-vous pour répondre.